Click here to Skip to main content
15,906,625 members

Comments by Ansas Bogdan (Top 1 by date)

Ansas Bogdan 24-Feb-11 12:13pm View    
Deleted
/*So this is you final answer? Then could you provide me with a sample where this actually fails to proof that you are correct? Or are you just unable to lose a debate ?.*/

Ok i admid my previous version didnt work in certain cases. But this actually even works in that cases and scince the compiler does the same i dont think this way would ever fail.